ChaseDream
搜索
返回列表 发新帖
查看: 1803|回复: 5
打印 上一主题 下一主题

请教LSAT中一种逻辑的问法

[复制链接]
楼主
发表于 2004-5-17 10:49:00 | 只看该作者

请教LSAT中一种逻辑的问法

Which one of the following, if true, most calls into question the XXX’argument?


我理解成削弱,结果答案总是做错,搞不清楚到底是加强还是削弱XXX的观点。


For example:


The gray squirrel, introduced into local woodlands ten years ago, threatens the indigenous population of an endangered owl species, because the squirrels’ habitual stripping of tree bark destroys the trees in which the owls nest. Some local officials have advocated setting out poison for the gray squirrels. The officials argue that this measure, while eliminating the squirrels, would pose no threat to the owl population, since the poison would be placed in containers accessible only to squirrels and other rodents.


Which one of the following, if true, most calls into question the officials’ argument?


(A) One of the species whose members are likely to eat the poison is the red squirrel, a species on which owls do not prey.


(B) The owls whose nesting sites are currently being destroyed by the gray squirrels feed primarily on rodents.


(C) No indigenous population of any other bind species apart from the endangered owls is threatened by the gray squirrels.


(D) The owls that tare threatened build their nests in the tops of trees, but the gray squirrels strip away back from the trunks.


(E) The officials’ plan entails adding the poison to food sources that are usually eaten by rodents but not by other animals.



沙发
发表于 2004-5-17 11:31:00 | 只看该作者

削弱

板凳
 楼主| 发表于 2004-5-17 15:08:00 | 只看该作者

谢谢你的回答,这道题是削弱的话,应该怎么去削弱;the officials’ argument 是this measure, while eliminating the squirrels, would pose no threat to the owl population, since the poison would be placed in containers accessible only to squirrels and other rodents。削弱要么非因同果,要么同因异果。可是这道题怎么也得不出答案b,请高手指教。

地板
 楼主| 发表于 2004-5-17 15:40:00 | 只看该作者
还没人来回答,我顶一下!
5#
发表于 2004-5-18 09:39:00 | 只看该作者

措施目的型削弱,能证明措施无效或副作用巨大,就是削弱了。本题目的是保护owl, 可是措施的结果虽然消灭了grey squirrel, 可是将owl的食物Rodents也都消灭了,没法儿活了!

削弱要么非因同果,要么同因异果。可是这道题怎么也得不出答案b,请高手指教。

太高深,没明白意思。

6#
 楼主| 发表于 2004-5-18 10:03:00 | 只看该作者
嗯,现在明白了,文中the officials’ argument的推倒结构是the poison would be placed in containers accessible only to squirrels and other rodents推出this measure would pose no threat to the owl population。而b答案中说the owls feed primarily on rodents。从而导致this measure would pose  threat to the owl population。相同的原因不同的结果。呵呵,没有认真审题。谢谢!
您需要登录后才可以回帖 登录 | 立即注册

Mark一下! 看一下! 顶楼主! 感谢分享! 快速回复:

手机版|ChaseDream|GMT+8, 2024-10-6 22:25
京公网安备11010202008513号 京ICP证101109号 京ICP备12012021号

ChaseDream 论坛

© 2003-2023 ChaseDream.com. All Rights Reserved.

返回顶部